× Indicații, rezolvări și soluții pentru problemele și exercițiile de liceu.

file Întrebare Matrice

Mai Mult
acum 7 ani 7 luni #744 de ildo
ildo a creat subiectul: Matrice
Pentru orice matrice A si B din\[M_{3}\left ( R \right )\]
notam
\[\left [ A,B \right ]=AB-BA\]
Sa se arate ca
\[\left [ A,B \right ]\neq I_{3}\]

Vă rugăm Autentificare sau Crează un cont să participaţi la discuţie.

Mai Mult
acum 7 ani 7 luni - acum 7 ani 7 luni #745 de gordianknot
gordianknot a răspuns subiectului: Matrice
Sa presupunem ca exista \(A,B\in M_{3}\left ( \mathbb{R} \right )\) astfel incat \(AB-BA=I_{3}\).

Se cunosc urmatoarele:

\(Tr\left ( AB \right )=Tr\left ( BA \right )\) si \(Tr\left ( A+B \right )=Tr\left ( A \right )+Tr\left ( B \right )\), \(\forall A,B\in M_{3}\left ( \mathbb{R} \right )\)
(\(Tr\left ( A \right )=a_{11}+a_{22}+a_{33}\) - urma matricei \(A\) ).

Avem, deci

\(Tr\left ( AB -BA\right )=Tr\left ( AB \right )-Tr\left ( BA \right )=0\).

Atunci, in egalitatea \(AB-BA=I_{3}\):

- pe partea stanga avem o matrice cu urma \(0\) (\(Tr\left ( AB-BA \right )=0\) ),

- pe partea dreapta o matrice cu urma \(3\) (\(Tr\left ( I_{3} \right )=1+1+1=3\neq 0\) ) - contradictie, presupunerea nu a fost corecta, deci \([A,B]\neq I_{3},\: \forall A,B\in M_{3}\left ( \mathbb{R} \right )\).

Observatie: Egalitatea \([A,B]\neq I_{n}\), este adevarata \( \: \forall A,B\in M_{n}\left ( \mathbb{R} \right )\) ,\( n\in \mathbb{N},\: n\geq 2\) (demonstratia e similara cu cea de sus).
Ultima Editare: acum 7 ani 7 luni de gordianknot. Motiv: Completare
Următorul utilizator(ori) v-au spus Mulțumesc: ildo

Vă rugăm Autentificare sau Crează un cont să participaţi la discuţie.

  • Nepermis: pentru a crea subiect nou.
  • Nepermis: pentru a răspunde.
  • Nepermis: pentru a adăuga atașamente.
  • Nepermis: să-ți editeze mesajele.
Timp creare pagină: 0.157 secunde
Motorizat de Forum Kunena